100% satisfaction guarantee Immediately available after payment Both online and in PDF No strings attached
logo-home
Fundamentals of Insurance Planning Practice Exam| QUESTIONS WITH COMPLETE SOLUTIONS $13.49   Add to cart

Exam (elaborations)

Fundamentals of Insurance Planning Practice Exam| QUESTIONS WITH COMPLETE SOLUTIONS

 1 view  0 purchase
  • Course
  • Institution

A loss exposure is defined as: (A) a loss that might occur (B) an undesirable end result of a risk (C) the largest possible loss (D) the probability of a loss The answer is (A) by definition Which of the following statements concerning the beneficiary provisions in a life insurance poli...

[Show more]

Preview 4 out of 40  pages

  • November 5, 2022
  • 40
  • 2022/2023
  • Exam (elaborations)
  • Questions & answers
avatar-seller
Fundamentals of Insurance Planning Practice Exam| QUESTIONS
WITH COMPLETE SOLUTIONS
A loss exposure is defined as:
(A) a loss that might occur
(B) an undesirable end result of a risk
(C) the largest possible loss
(D) the probability of a loss Correct Answer: The answer is (A) by definition

Which of the following statements concerning the beneficiary provisions in a life insurance
policy is correct?
(A) Only one person may be named as a primary beneficiary, but several may be named as
contingent beneficiaries.
(B) The primary beneficiary's share of policy proceeds is generally reduced if the contingent
beneficiary outlives the insured.
(C) The estate of the insured is commonly named as beneficiary to avoid transfer taxes.
(D) The contingent beneficiary will receive policy proceeds only if the primary beneficiary
predeceases the insured. Correct Answer: The answer is(D).
(A) is incorrect because one or more people or entities may be named as either primary or
contingent beneficiaries.
(B) is incorrect because the contingent beneficiary receives nothing and all proceeds go the
primary beneficiary when the insured dies unless the primary beneficiary predeceases the insured
or loses entitlement to the benefits for some other reason.
(C) is incorrect because transfer taxes may be incurred when the estate is the beneficiary.

Which of the following types of insurance is often sold by mortgage lenders to provide the funds
necessary to pay off a 10-year mortgage loan if the insured dies?
(A) decreasing term
(B) increasing term
(C) modified whole life
(D) 10-year renewable term Correct Answer: The answer is (A).
(B) is incorrect because the amount of insurance under an increasing term policy increases while
the amount needed to pay off the mortgage decreases.
(C) is incorrect because the amount of insurance under a modified whole life policy remains
constant while the amount needed to pay off the mortgage decreases.
(D) is incorrect because the amount of insurance under a 10-year renewable term policy remains
constant for 10 years while the amount needed to pay off the mortgage decreases.

Which of the following is most likely to be a condition that will result in the payment of
unemployment compensation benefits?
(A) unemployment because of a labor dispute
(B) unemployment as a result of voluntarily leaving a job without good cause
(C) unemployment resulting from discharge that was prompted by misconduct
(D) unemployment that results from adverse economic conditions Correct Answer: The answer
is (D).

,(A), (B), and (C) are incorrect because common reasons for disqualification under
unemployment compensation programs include involvement in a labor dispute, voluntarily
leaving a job without good cause, and discharge for misconduct.

Which of the following statements concerning inpatient hospital care under Part A of Medicare is
correct?
(A) There is a $200 annual deductible.
(B) Benefits are paid in full after the 60th day of hospitalization.
(C) There is a lifetime limit on the number of days of treatment in psychiatric hospitals.
(D) There is coverage for up to 210 days of care in each benefit period. Correct Answer: The
answer is (C).
(A) is incorrect because hospital benefits are not subject to an annual deductible. However, there
is a benefit period deductible, and it is much higher than $200.
(B) is incorrect because benefits are paid in full for only the first 60 days in each benefit period,
subject to the initial deductible.
(D) is incorrect because Part A pays for hospital services in full for up to 60 days in each benefit
period after the deductible has been met. It provides benefits for an additional 30 days subject to
a daily patient copayment. There is also a lifetime reserve of 60 additional days, but once these
days are used, they are not available for future benefit periods.

Insurance policies that give the policyowner the right to renew coverage but do not guarantee
future rates are referred to as
(A) cancelable
(B) guaranteed renewable
(C) noncancelable
(D) optionally renewable Correct Answer: The answer is (B).
(A) is incorrect because the insurer has the right to cancel a cancelable policy during the period
for which premiums have been paid.
(C) is incorrect because future rates are guaranteed in a noncancelable policy.
(D) is incorrect because the insurer has a right to refuse to
renew an optionally renewable policy.

The law of large numbers briefly states that as the
(A) number of independent events increases, the likelihood increases that the actual results will
be close to the expected results
(B) degree of reliability placed on past experience increases, the likelihood increases that the
expected results will be close to the past results
(C) uncertainty surrounding the expected results decreases, the likelihood decreases that the
range of outcomes will vary widely
(D) average size of each loss varies from event to event, the likelihood increases that the next
event will be close to the average Correct Answer: The answer is (A).
(B), (C), and (D) are incorrect because the law of large numbers states that as the number of
independent events increases, the likelihood increases that the actual results will be close to the
expected results.

,Which of the following statements concerning the elimination period in a long-term care
insurance policy is correct?
(A) Most comprehensive policies have separate elimination periods for facility care and home
health care.
(B) A tax-qualified policy must have an elimination period of at least 90 days.
(C) The effect of the elimination period is to reduce a policy's maximum benefit period.
(D) The applicant typically selects the length of the elimination period from three to five
available options. Correct Answer: The answer is (D). The applicant typically selects the length
of the elimination period from three to five available options.
(A) is incorrect because most comprehensive long-term care insurance policies have a single
elimination period that can be met with a combination of days from when an insured is in a long-
term care facility or receiving home health care services.
(B) is incorrect because a tax-qualified policy can have an elimination period of any length of
time, assuming it is acceptable under state law. The only 90-day requirement is that the insured
be certified as being unable to perform the required number of ADLs for at least 90 days.
(C) is incorrect because the elimination period only postpones the date until benefits start. Once
they start, the maximum benefit period applies.

The standard amount of insurance for loss of use (Coverage D) under an unendorsed HO-3
policy is what percentage of the amount of insurance for the dwelling (Coverage A)?
(A) 10 percent
(B) 20 percent
(C) 30 percent
(D) 50 percent Correct Answer: The answer is (C).
The standard limit for Coverage D is 30 percent of the Coverage A limit. Prior to 2000, the limit
was 20 percent. The limit for Coverage B (other structures) is 10 percent of the Coverage A
limit, while the limit for Coverage C (personal property) is 50 percent of the Coverage A limit.

Ken has a personal auto policy (PAP) with coverage limits of $500,000 for liability (Part A),
$15,000 for medical payments (Part B), $50,000 for uninsured motorists (Part C), and no
underinsured motorists endorsement. If Ken receives $100,000 in bodily injuries as a result of an
auto accident with another driver who has only the minimum state requirement of $35,000 of
liability coverage, how would the two policies cover Ken's injuries if the other driver was at
fault?
(A) Ken's PAP would cover the entire $100,000 under Part A because the other driver's coverage
is inadequate to compensate Ken for his injuries.
(B) The other driver's policy would pay its limit of $35,000 while Ken's PAP would pay $15,000
under Part B and $50,000 under Part C.
(C) The other driver's policy would pay its limit of $35,000 while Ken's PAP would pay $50,000
under Part C with the remaining $15,000 not covered by either policy.
(D) The other driver's policy would pay its limit of $35,000 while Ken's PAP would pay $15,000
under Part B with the remaining $50,000 not covered by either policy. Correct Answer: The
answer is (D).
Ken would collect $35,000 from the other party's liability insurer. His own policy would also pay
the full $15,000 medical payment limit. There is no coverage for the remaining $50,000 because
Ken does not have coverage for underinsured motorists.

, King purchased an insurance policy that will pay $200 for each day he is hospitalized, regardless
of the amount billed by the hospital. This insurance policy is considered which of the following?
(A) a bilateral contract
(B) a contract of indemnity
(C) a valued contract
(D) an impersonal contract Correct Answer: The answer is (C).
(A) is incorrect because insurance is a unilateral contract.
(B) is incorrect because payments under a contract of indemnity reflect the amount of the loss.
(D) is incorrect because insurance is a personal contract.

William is covered under his employer's major medical expense plan with a $1 million lifetime
limit. The plan has a calendar-year deductible of $1,000, a 75 percent coinsurance provision that
applies to the next $8,000 of covered expenses, and full coverage for any remaining covered
expenses. If William incurs covered medical expenses of $15,000 during the year, how much
will be paid by his employer's plan?
(A) $10,500
(B) $12,000
(C) $13,000
(D) $14,000 Correct Answer: The answer is (B).
His employer's plan will pay $12,000. If William incurs $15,000 of covered medical expenses,
he must pay $1,000 out-of-pocket to meet the deductible. Of the remaining $14,000 of expenses,
$8,000 of it is subject to the 75 percent coinsurance provision. This means that William must pay
another $2,000 (.25 × $8,000 = $2,000) out-of-pocket for a total amount of $3,000 ($1,000
deductible + $2,000 coinsurance = $3,000). Subtracting $3,000 from $15,000 leaves $12,000 for
the medical expense plan to cover.

Which of the following statements describes factors that should be considered in the argument
whether it is better to buy term life insurance or to buy permanent insurance and invest the
premium difference?
(A) The cash value of a life insurance policy can readily be liquidated.
(B) People are less likely to pay a life insurance premium than they are to make a voluntary
investment.
(C) The life insurance industry has a questionable solvency record.
(D) Increases in life insurance cash values are subject to federal income taxes as they accrue.
Correct Answer: The answer is (A).
(B) is incorrect because people are more likely to pay a life insurance premium than they are to
make a voluntary investment.
(C) is incorrect because the life insurance industry has a solvency record unmatched by any other
type of business organization.
(D) is incorrect because increases in cash values are not subject to federal income taxes as they
accrue, in contrast to the earnings in a separate investment program, which are often taxed as
ordinary income.

Susan has a personal auto policy (PAP) with a liability limit of $500,000. She also has a personal
umbrella policy with a limit of $1 million and a self-insured retention (SIR) of

The benefits of buying summaries with Stuvia:

Guaranteed quality through customer reviews

Guaranteed quality through customer reviews

Stuvia customers have reviewed more than 700,000 summaries. This how you know that you are buying the best documents.

Quick and easy check-out

Quick and easy check-out

You can quickly pay through credit card or Stuvia-credit for the summaries. There is no membership needed.

Focus on what matters

Focus on what matters

Your fellow students write the study notes themselves, which is why the documents are always reliable and up-to-date. This ensures you quickly get to the core!

Frequently asked questions

What do I get when I buy this document?

You get a PDF, available immediately after your purchase. The purchased document is accessible anytime, anywhere and indefinitely through your profile.

Satisfaction guarantee: how does it work?

Our satisfaction guarantee ensures that you always find a study document that suits you well. You fill out a form, and our customer service team takes care of the rest.

Who am I buying these notes from?

Stuvia is a marketplace, so you are not buying this document from us, but from seller Classroom. Stuvia facilitates payment to the seller.

Will I be stuck with a subscription?

No, you only buy these notes for $13.49. You're not tied to anything after your purchase.

Can Stuvia be trusted?

4.6 stars on Google & Trustpilot (+1000 reviews)

85169 documents were sold in the last 30 days

Founded in 2010, the go-to place to buy study notes for 14 years now

Start selling
$13.49
  • (0)
  Add to cart